LSAT and Law School Admissions Forum

Get expert LSAT preparation and law school admissions advice from PowerScore Test Preparation.

User avatar
 Dave Killoran
PowerScore Staff
  • PowerScore Staff
  • Posts: 5850
  • Joined: Mar 25, 2011
|
#88230
Complete Question Explanation
(The complete setup for this game can be found here: lsat/viewtopic.php?f=174&p=88228#p88228)

The correct answer choice is (B).

The question stem asks you to supply a piece of information that will result in only one viable solution. The key here is to be careful using the templates with this question, because it is easy to fall into the trap of thinking that if a template has only one unplaced variable, placing that variable will result in that solution. A safer approach is to consider each variable:

X: ..... X is already placed by the first rule, and thus placing X will not result in a single solution. Hence, answer choice (D) can be eliminated.
Y: ..... Y is part of the ZY block, and we have already shown that the block has two placement options, and neither option forces just one ..... ..... ..... solution. Therefore, Y is not a likely candidate to be part of the correct answer.
Z: ..... Z is part of the ZY block, and we have already shown that the block has two placement options, and neither option forces just one ..... ..... ..... solution. Thus, answer choice (E) can be eliminated.
W: ..... The only role that W plays is as the necessary condition in the final rule. Just as making the necessary condition occur does not result in ..... ..... any inference (don’t make a Mistaken Reversal!), placing W in this game does not result in a single solution. Thus, answer choice (C) ..... ..... can be eliminated.
T: ..... T is a powerful variable but only when it signs with agency S. As a sufficient condition, when T signs with S, then W must sign with S. As X ..... ..... is already signed with F, the ZY block must then sign with P, resulting in just one solution. Thus, answer choice (B) is correct. The ..... ..... other answer choice involving T, answer choice (A), is incorrect because when T signs with F, nothing must occur as a result, and ..... ..... ..... therefore a single solution does not result from T signing with F.
 angelsfan0055
  • Posts: 39
  • Joined: Feb 26, 2021
|
#86761
Would appreciate some guidance on this question (and No. 21 in this LG section, too)
 Jeremy Press
PowerScore Staff
  • PowerScore Staff
  • Posts: 1000
  • Joined: Jun 12, 2017
|
#86790
Hi angelsfan,

Since this thread is for question #4 in this section, I'll limit my comments to that question. If you could post a question about #21 on that question's thread, we'll answer it over there.

With Justify style questions (questions that ask for an answer that leads to a specific result, in this case an answer that leads to a "completely determined" solution), we're often forced to test an answer choice or two, in order to confirm that the result of applying the answer is "complete." However, I'd like to be able to take an educated guess about which answer (or answers) to test, so that I'm not stuck just randomly testing my way through 4 or 5 answer choices.

What would be a good answer to test?
  • An answer that has the strongest impact on the game, i.e. that sets off the most dominoes, getting me to a completely determined solution.
  • If you created templates (which would be a great approach to this game, given that the YZ block can only go with either the Star Agency or the Premier Agency), you could also quickly look for an answer that, when plugged into the templates, allows no further "wiggle room" in that template's diagram
Both of those considerations counsel in favor of testing answer choice B.
  • Putting Traugott in Star Agency has a strong impact on the game, because it immediately sets off the "domino" of also forcing West into Star Agency (from the last rule).
  • If you built templates for this game, you'll see both that there is only one available template where Traugott can be placed in Star Agency, and that in that template everything else is already determined, so placing T into Star fills out the last piece of information required to fully solve it
When you test answer choice B, you do find that the game is fully solved, so you're done with the question. Walking quickly through the steps of that: With T in Star, W is also placed in Star (from the last rule in the game). X is always in Fame (and Z and Y are not, because of the combination of the second and third rules). Since Z and Y have to be together, and somebody has to be in Premier, this means that Z and Y must be in Premier, and the game is fully solved.

How would I avoid the other answer choices? None of the other answer choices include a variable placement that, from the listed rules, immediately forces me to place another variable. And none of the other answer choices fits into a single template where every other variable's placement is fully determined.

I hope this helps!
 hope
  • Posts: 84
  • Joined: Feb 13, 2018
|
#92949
Dave Killoran wrote: Sat Apr 24, 2021 8:41 am Complete Question Explanation
(The complete setup for this game can be found here: lsat/viewtopic.php?f=174&p=88228#p88228)

The correct answer choice is (B).

The question stem asks you to supply a piece of information that will result in only one viable solution. The key here is to be careful using the templates with this question, because it is easy to fall into the trap of thinking that if a template has only one unplaced variable, placing that variable will result in that solution. A safer approach is to consider each variable:

X: ..... X is already placed by the first rule, and thus placing X will not result in a single solution. Hence, answer choice (D) can be eliminated.
Y: ..... Y is part of the ZY block, and we have already shown that the block has two placement options, and neither option forces just one ..... ..... ..... solution. Therefore, Y is not a likely candidate to be part of the correct answer.
Z: ..... Z is part of the ZY block, and we have already shown that the block has two placement options, and neither option forces just one ..... ..... ..... solution. Thus, answer choice (E) can be eliminated.
W: ..... The only role that W plays is as the necessary condition in the final rule. Just as making the necessary condition occur does not result in ..... ..... any inference (don’t make a Mistaken Reversal!), placing W in this game does not result in a single solution. Thus, answer choice (C) ..... ..... can be eliminated.
T: ..... T is a powerful variable but only when it signs with agency S. As a sufficient condition, when T signs with S, then W must sign with S. As X ..... ..... is already signed with F, the ZY block must then sign with P, resulting in just one solution. Thus, answer choice (B) is correct. The ..... ..... other answer choice involving T, answer choice (A), is incorrect because when T signs with F, nothing must occur as a result, and ..... ..... ..... therefore a single solution does not result from T signing with F.
Hi Dave. I understand how B is correct. But doesn't E also work?

X T
Z W
Y

As the necessary condition, W could be placed into W without T having to be there also. That way, the template is completely determined. What am I not seeing? Thanks and Happy 2022! :-?
 hope
  • Posts: 84
  • Joined: Feb 13, 2018
|
#92950
Dave Killoran wrote: Sat Apr 24, 2021 8:41 am Complete Question Explanation
(The complete setup for this game can be found here: lsat/viewtopic.php?f=174&p=88228#p88228)

The correct answer choice is (B).

The question stem asks you to supply a piece of information that will result in only one viable solution. The key here is to be careful using the templates with this question, because it is easy to fall into the trap of thinking that if a template has only one unplaced variable, placing that variable will result in that solution. A safer approach is to consider each variable:

X: ..... X is already placed by the first rule, and thus placing X will not result in a single solution. Hence, answer choice (D) can be eliminated.
Y: ..... Y is part of the ZY block, and we have already shown that the block has two placement options, and neither option forces just one ..... ..... ..... solution. Therefore, Y is not a likely candidate to be part of the correct answer.
Z: ..... Z is part of the ZY block, and we have already shown that the block has two placement options, and neither option forces just one ..... ..... ..... solution. Thus, answer choice (E) can be eliminated.
W: ..... The only role that W plays is as the necessary condition in the final rule. Just as making the necessary condition occur does not result in ..... ..... any inference (don’t make a Mistaken Reversal!), placing W in this game does not result in a single solution. Thus, answer choice (C) ..... ..... can be eliminated.
T: ..... T is a powerful variable but only when it signs with agency S. As a sufficient condition, when T signs with S, then W must sign with S. As X ..... ..... is already signed with F, the ZY block must then sign with P, resulting in just one solution. Thus, answer choice (B) is correct. The ..... ..... other answer choice involving T, answer choice (A), is incorrect because when T signs with F, nothing must occur as a result, and ..... ..... ..... therefore a single solution does not result from T signing with F.
NEVER MIND DAVE. DISREGARD MY POST. I GOT IT NOW. THANKS.

Get the most out of your LSAT Prep Plus subscription.

Analyze and track your performance with our Testing and Analytics Package.